Bạn chưa đăng nhập. Vui lòng đăng nhập để hỏi bài

Những câu hỏi liên quan
Trúc Mai Huỳnh
Xem chi tiết
๖ۣۜLuyri Vũ๖ۣۜ
Xem chi tiết
Kiệt Nguyễn
11 tháng 10 2020 lúc 10:08

Ta có: \(P=\frac{\sqrt{yz}}{x+2\sqrt{yz}}+\frac{\sqrt{zx}}{y+2\sqrt{zx}}+\frac{\sqrt{xy}}{z+2\sqrt{xy}}=\frac{1}{\frac{x}{\sqrt{yz}}+2}+\frac{1}{\frac{y}{\sqrt{zx}}+2}+\frac{1}{\frac{z}{\sqrt{xy}}+2}\)

Đặt \(\frac{x}{\sqrt{yz}}=c,\frac{y}{\sqrt{zx}}=t;\frac{z}{\sqrt{xy}}=k\left(c,t,k>0\right)\)thì ctk = 1

Ta cần tìm giá trị lớn nhất của \(P=\frac{1}{c+2}+\frac{1}{t+2}+\frac{1}{k+2}\)với ctk = 1

Dự đoán MaxP = 1 khi c = t = k = 1

Thật vậy: \(P=\frac{kt+2k+2t+4+ct+2c+2t+4+ck+2c+2k+4}{\left(c+2\right)\left(t+2\right)\left(k+2\right)}=\frac{\left(kt+tc+ck\right)+4\left(c+t+k\right)+12}{ctk+2\left(kt+tc+ck\right)+4\left(c+t+k\right)+8}\le\frac{\left(kt+tc+ck\right)+4\left(c+t+k\right)+12}{1+\left(kt+tc+ck\right)+3\sqrt[3]{\left(ctk\right)^2}+4\left(c+t+k\right)+8}=1\)Đẳng thức xảy ra khi x = y = z

Khách vãng lai đã xóa
Edogawa Conan
11 tháng 10 2020 lúc 10:13

Ta có: \(\frac{\sqrt{yz}}{x+2\sqrt{yz}}=\frac{1}{2}\left(1-\frac{x}{x+2\sqrt{yz}}\right)\le\frac{1}{2}\left(1-\frac{x}{x+y+z}\right)=\frac{1}{2}\left(\frac{y+z}{x+y+z}\right)\)(bđt cosi) (1)

CMTT: \(\frac{\sqrt{xz}}{y+2\sqrt{xz}}\le\frac{1}{2}\left(\frac{x+z}{x+y+z}\right)\)(2)

\(\frac{\sqrt{xy}}{z+2\sqrt{xy}}\le\frac{1}{2}\left(\frac{x+y}{x+y+z}\right)\)(3)

Từ (1), (2) và (3) cộng vế theo vế ta có:

\(\frac{\sqrt{yz}}{x+2\sqrt{yz}}+\frac{\sqrt{xz}}{y+2\sqrt{xz}}+\frac{\sqrt{xy}}{z+2\sqrt{xy}}\le\frac{1}{2}\left(\frac{y+z}{x+y+z}\right)+\frac{1}{2}\left(\frac{x+z}{x+y+z}\right)+\frac{1}{2}\left(\frac{x+y}{x+y+z}\right)\)

=> P \(\le\frac{1}{2}\left(\frac{y+z+x+z+x+y}{x+y+z}\right)=\frac{1}{2}\cdot\frac{2\left(x+y+z\right)}{x+y+z}=1\)

Dấu "=" xảy ra <=> x = y = z

Vậy MaxP = 1 <=> x = y = z

Khách vãng lai đã xóa
Trí Tiên亗
11 tháng 10 2020 lúc 10:19

một bài khá hay :)

Ta có \(\frac{\sqrt{yz}}{x+2\sqrt{yz}}=1-\frac{x}{x+2\sqrt{yz}}\le1-\frac{x}{x+y+z}\left(1\right)\)

 Tương tự \(\frac{\sqrt{xz}}{y+2\sqrt{xz}}=1-\frac{y}{y+2\sqrt{xz}}\le1-\frac{y}{x+y+z}\left(2\right)\)

\(\frac{\sqrt{xy}}{z+2\sqrt{xy}}=1-\frac{z}{z+2\sqrt{xy}}\le1-\frac{z}{x+y+z}\left(3\right)\)

Cộng (1);(2);(3)

\(2P\le3-\frac{x+y+z}{x+y+z}=2\Rightarrow P\le1\)

Vậy \(minP=1\)Khi và chỉ khi \(x=y=z\)

Khách vãng lai đã xóa
Nhái Channel
Xem chi tiết
Thăng Vũ
21 tháng 11 2018 lúc 21:18

Sử dụng Cô-si ngược dấu có thêm hằng số

Thăng Vũ
21 tháng 11 2018 lúc 21:22

Kq là 1 nhé

Nhái Channel
21 tháng 11 2018 lúc 22:06

viết cách làm giúp mk vs

Hồ Minh Phi
Xem chi tiết
Akai Haruma
20 tháng 11 2018 lúc 0:55

Lời giải:

Để cho gọn đặt \((\sqrt{x}; \sqrt{y}; \sqrt{z})=(a,b,c)\) với \(a,b,c>0\)

Khi đó:

\(A=\frac{bc}{a^2+2bc}+\frac{ac}{b^2+2ac}+\frac{ab}{c^2+2ab}\)

\(=\frac{1}{2}(\frac{2bc}{a^2+2bc}+\frac{2ac}{b^2+2ac}+\frac{2ab}{c^2+2ab})\)

\(=\frac{1}{2}\left(1-\frac{a^2}{a^2+2bc}+1-\frac{b^2}{b^2+2ac}+1-\frac{c^2}{c^2+2ab}\right)\)

\(=\frac{3}{2}-\frac{1}{2}\underbrace{\left(\frac{a^2}{a^2+2bc}+\frac{b^2}{b^2+2ac}+\frac{c^2}{c^2+2ab}\right)}_{M}\)

Áp dụng BĐT Cauchy-Schwarz:

\(M\geq \frac{(a+b+c)^2}{a^2+2bc+b^2+2ac+c^2+2ab}=\frac{(a+b+c)^2}{(a+b+c)^2}=1\)

\(\Rightarrow A=\frac{3}{2}-\frac{1}{2}M\leq \frac{3}{2}-\frac{1}{2}=1\)

Vậy \(A_{\max}=1\Leftrightarrow a=b=c\Leftrightarrow x=y=z\)

Fire Sky
Xem chi tiết
Love Phương Forever
1 tháng 5 2019 lúc 15:52

Quẩy lên các em êii

Phùng Minh Quân
1 tháng 5 2019 lúc 16:04

\(A=\frac{\sqrt{xy}}{z+2\sqrt{xy}}+\frac{\sqrt{yz}}{x+2\sqrt{yz}}+\frac{\sqrt{zx}}{y+2\sqrt{zx}}\)

\(2A=\frac{z+2\sqrt{xy}}{z+2\sqrt{xy}}-\frac{z}{z+2\sqrt{xy}}+\frac{x+2\sqrt{yz}}{x+2\sqrt{yz}}-\frac{x}{x+2\sqrt{yz}}+\frac{y+2\sqrt{zx}}{y+2\sqrt{zx}}-\frac{y}{y+2\sqrt{zx}}\)

\(=3-\left(\frac{x}{x+2\sqrt{yz}}+\frac{y}{y+2\sqrt{zx}}+\frac{z}{z+2\sqrt{xy}}\right)\le3-\left(\frac{x}{x+y+z}+\frac{y}{x+y+z}+\frac{z}{x+y+z}\right)\)

\(=3-\frac{x+y+z}{x+y+z}=3-1=2\)\(\Leftrightarrow\)\(A\le\frac{2}{2}=1\)

Dấu "=" xảy ra \(\Leftrightarrow\)\(x=y=z\)

...

Lê Huỳnh
Xem chi tiết
Lê Huỳnh
Xem chi tiết
Phước Nguyễn
23 tháng 4 2016 lúc 21:15

Bạn ghi sai đề rồi nhé! Nếu ta lần lượt thay số vào các biến  \(x,y,z\) ở vế trái của bất đẳng thức trên (chẳng hạng như  \(\frac{1}{3}\)) kết hợp với chú ý rằng \(x=y=z\)  (sẽ được chứng minh ở các bước sau này), khi đó kết quả sẽ cho ra khác, tức là  \(\frac{3}{\sqrt{2}}\) (vô lý!). Đó là lý do mình phải 'viết lại' đề cộng với một chút chỉnh sửa hợp lý về phương diện toán học. Hmmm, vất vả vật lộn với bài này quá nya. \(3\)  \(s\) đi!

Đề: Cho ba số thực dương  \(x,y,z\)  thỏa mãn  \(x+y+z=1\)  

Chứng minh rằng: \(\sqrt{\frac{xy}{z+xy}}+\sqrt{\frac{yz}{x+yz}}+\sqrt{\frac{xz}{y+yz}}\le\frac{3}{2}\)  \(\left(\text{*}\right)\)

Lời giải:

Từ giả thiết đã cho ở trên, ta dễ dàng chứng minh được  \(1>x,y,z>0\)  với mọi  \(x,y,z\in R^+\)

\(\Rightarrow\)  \(1-x>0;\)  \(1-y>0;\)  \(1-z>0\)  

Khi đó, áp dụng bất đẳng thức  \(AM-GM\)  cho hai số không âm với chú ý rằng  \(x+y+z=1\)  (theo giả thiết), ta có: 

\(\sqrt{\frac{xy}{z+xy}}=\sqrt{\frac{xy}{1-x-y+xy}}=\sqrt{\frac{xy}{\left(1-x\right)\left(1-y\right)}}\le\frac{1}{2}\left(\frac{x}{1-y}+\frac{y}{1-x}\right)\)  \(\left(1\right)\)

Hoàn toàn tương tự với vòng hoán vị  \(y\)  \(\rightarrow\)  \(z\)  \(\rightarrow\)  \(x\), ta chứng minh được:

\(\sqrt{\frac{yz}{x+yz}}\le\frac{1}{2}\left(\frac{y}{1-z}+\frac{z}{1-y}\right)\)  \(\left(2\right)\)  và  \(\sqrt{\frac{xz}{y+xz}}\le\frac{1}{2}\left(\frac{z}{1-x}+\frac{x}{1-z}\right)\)  \(\left(3\right)\)

Cộng từng vế các bất đẳng thức \(\left(1\right);\)  \(\left(2\right);\)  và  \(\left(3\right),\)  ta được:

\(VT\left(\text{*}\right)\le\frac{1}{2}\left[\left(\frac{y}{1-x}+\frac{z}{1-x}\right)+\left(\frac{x}{1-y}+\frac{z}{1-y}\right)+\left(\frac{x}{1-z}+\frac{y}{1-z}\right)\right]=\frac{1}{2}\left(1+1+1\right)=\frac{3}{2}=VP\left(\text{*}\right)\)

Dấu  \("="\)  xảy ra  \(\Leftrightarrow\)  \(a=b=c=\frac{1}{3}\)

Trần Anh
23 tháng 4 2016 lúc 20:31

ở mẫu phải là dấu cộng mới đúng chứ bạn

Bi Bi
Xem chi tiết
Nguyễn Việt Lâm
16 tháng 10 2019 lúc 20:27

\(3-2P=\frac{x}{x+2\sqrt{yz}}+\frac{y}{y+2\sqrt{xz}}+\frac{z}{z+2\sqrt{xy}}\)

\(3-2P\ge\frac{x}{x+y+z}+\frac{y}{x+y+z}+\frac{z}{x+y+z}=1\)

\(\Rightarrow2P\le2\Rightarrow P\le1\)

Dấu "=" xảy ra khi \(x=y=z\)

\(M\le\sqrt{\left(1+1\right)\left(x+y+2\right)}=\sqrt{20}=4\sqrt{5}\)

\(M_{max}=4\sqrt{5}\) khi \(\left\{{}\begin{matrix}x-2=y+4\\x+y=8\end{matrix}\right.\) \(\Rightarrow\left\{{}\begin{matrix}x=7\\y=1\end{matrix}\right.\)

Nguyen Duy Dai
Xem chi tiết